Xem bài viết đơn
Old 11-07-2012, 01:56 AM   #3
Shuichi Akai
+Thành Viên+
 
Tham gia ngày: Nov 2011
Bài gởi: 12
Thanks: 40
Thanked 10 Times in 8 Posts
Mình làm giống như ý tưởng bạn ở trên
AM - GM cho k số:
$(1+a_k)^k=\left (\dfrac{1}{k-1}.(k-1)+a_k\right )^k\ge \dfrac{a_k}{(k-1)^{k-1}}.k^k $
từ đấy suy ra
$\prod (1+a_i)^i\geq a_2a_3...a_n\frac{2^2.3^3...n^n}{1^1.2^2...(n-1)^{n-1}}=n^n $
dấu bằng xảy ra $\Leftrightarrow a_i = \frac{1}{i-1} \Rightarrow a_2a_3...a_n < 1 $ (mâu thuẫn)
Vậy ta có đpcm
[RIGHT][I][B]Nguồn: MathScope.ORG[/B][/I][/RIGHT]
 
Shuichi Akai is offline   Trả Lời Với Trích Dẫn
 
[page compression: 7.14 k/8.10 k (11.85%)]